Induction proof: 1^3+2^3+...+n^3=(1+2+3+...+n)^2

Aion

Junior Member
Joined
May 8, 2018
Messages
91
Hi. if you find any fault in this proof please let me know since I couldn't find an answer on the back of my math book.

Prove that

\(\displaystyle 1^3 + 2^3 + ... + n^3 = (1+2+3+...+n)^2\)

Statement

Let \(\displaystyle P(n)\) be the statement \(\displaystyle \displaystyle\sum_{i=0}^{n} i^3 =1^3 + 2^3 + ... + n^3 = (1+2+3+...+n)^2\)

The sum of the first \(\displaystyle n\) positive integers is \(\displaystyle \frac{n(n+1)}{2}\) which is a theorem refered to as 1-1.

We can rewrite the statement \(\displaystyle P(n)\) based on theorem 1-1 as \(\displaystyle \displaystyle\sum_{i=0}^{n} i^3 = (\frac{n(n+1)}{2})^2\).

Now we will show that \(\displaystyle P(n)\) is true for all integers \(\displaystyle n \geq 0\)

Base case

\(\displaystyle P(0)\) is the statement \(\displaystyle \displaystyle\sum_{i=0}^{0} i^3 = (\frac{0(0+1)}{2})^2= 0\) which is clearly true.


The induction step

Let \(\displaystyle k \geq 0\) be an integer. Assume (for induction) that \(\displaystyle P(k)\) is true. That means \(\displaystyle \displaystyle\sum_{i=0}^{k} i^3\) \(\displaystyle \stackrel{?}{=}\)\(\displaystyle (\frac{k(k+1)}{2})^2\), is true for \(\displaystyle n=k\).

We will prove that \(\displaystyle P(k+1)\) is true as well. That is, we must prove that \(\displaystyle \displaystyle\sum_{i=0}^{k} i^3 + (k+1)^3= (\frac{(k+1)((k+1)+1)}{2})^2\). To prove this equation, start by adding \(\displaystyle (k+1)^3\)
to both sides of the inductive hypothesis:

\(\displaystyle \displaystyle\sum_{i=0}^{k+1} i^3= (\frac{k(k+1)}{2})^2 + (k+1)^3\). Now, simplifying the right side we get:

\(\displaystyle =\frac{k^2((k+1)^2)}{4} + (k+1)^3 = \frac{k^2(k+1^2) +(4k+4)(k+1)^2}{4} = \frac{(k+2)^2(k+1)^2}{2^2} = (\frac{((k+2)(k+1)=}{2})^2 = (\frac{(k+1)((k+1)+1)}{2})^2 \)

Thus \(\displaystyle P(k+1)\) is true, so by the principle of mathematical induction \(\displaystyle P(n)\) is true for all integers \(\displaystyle n \geq 0\)

Q.E.D.



 
Last edited:
Hi. if you find any fault in this proof please let me know since I couldn't find an answer on the back of my math book.

Prove that

\(\displaystyle 1^3 + 2^3 + ... + n^3 = (1+2+3+...+n)^2\)

Statement

\(\displaystyle \displaystyle\sum_{i=0}^{n} x^3 =1^3 + 2^3 + ... + n^3 = (1+2+3+...+n)^2\)

By using theorem 1-1: The sum of the first n positive integers is \(\displaystyle \frac{n(n+1)}{2}\)

We can rewrite the statement as \(\displaystyle \displaystyle\sum_{i=0}^{n} x^3 =1^3 + 2^3 + ... + n^3 = (1+2+3+...+n)^2=(\frac{n(n+1)}{2})^2\)

Show true for \(\displaystyle n=1\)

\(\displaystyle \displaystyle\sum_{i=0}^{1} x^3 = 0^3 + 1^3 = 1\) and \(\displaystyle 1^2 = 1\)

LHS(Left-hand side) = RHS(Right-hand side), therefore, the statement is true for \(\displaystyle n = 1\)

The induction hypothesis

\(\displaystyle \displaystyle\sum_{i=0}^{k+1} x^3\) \(\displaystyle \stackrel{?}{=}\) is true for some value of \(\displaystyle k≥n\), hence \(\displaystyle k≥1\).

The induction step

We must show that the statement is true for some value of \(\displaystyle n = k +1\), such that it will also be true for the next value of \(\displaystyle n\).


Assuming that \(\displaystyle \displaystyle\sum_{i=0}^{k} x^3=(\frac{k(k+1)}{2})^2\), we find that

\(\displaystyle \displaystyle\sum_{i=0}^{k+1} x^3= \displaystyle\sum_{i=0}^{k} x^3 + (k+1)^3 = (\frac{k(k+1)}{2})^2 + (k+1)^3= \)

\(\displaystyle \frac{k^2((k+1)^2)}{4} + (k+1)^2 = \frac{k^2(k+1^2) +(4k+4)(k+1)^2}{4} = \frac{(k+2)^2(k+1)^2}{2^2} = (\frac{(k+2)(k+1)}{2})^2 = (\frac{(k+1)((k+1)+1)}{2})^2) \)

The last equation is the same equation as \(\displaystyle (\frac{n(n+1)}{2})^2\) which is based on Theorem1-1, except that n is replaced by (k+1). Hence whenever it is true for all the integers 1,2,...,k, then it is true for the integer k+1.

The ideas are fine; but you have not written what you meant in some places.

First, you keep writing \(\displaystyle \displaystyle\sum_{i=0}^{n} x^3\). But that means \(\displaystyle \displaystyle\sum_{i=0}^{n} x^3 =x^3 + x^3 + ... + x^3\) ! Presumably you meant to write \(\displaystyle \displaystyle\sum_{i=0}^{n} i^3 = 0^3 + 1^3 + 2^3 + ... + n^3\).

The base case really should be n = 0, not n = 1, the way you've written the sum; you didn't state the conditions on n for the theorem, so if it starts at n = 1, you really should have written \(\displaystyle \displaystyle\sum_{i=1}^{n} i^3 = 1^3 + 2^3 + ... + n^3\).

Where you say you are stating the induction hypothesis, you are not really stating anything (an equal sign with a question mark doesn't assert anything). But the second line under "The Induction Step" is the induction hypothesis.

I could quibble about other parts of the wording, but I don't want to go too far. The important thing is that you got the math right; it's just the symbols and the words that need improvement ...
 
Hi. if you find any fault in this proof please let me know since I couldn't find an answer on the back of my math book.

Prove that

\(\displaystyle 1^3 + 2^3 + ... + n^3 = (1+2+3+...+n)^2\)

Statement

\(\displaystyle \displaystyle\sum_{i=0}^{n} x^3 =1^3 + 2^3 + ... + n^3 = (1+2+3+...+n)^2\)

By using theorem 1-1: (we don't know what Theorem 1-1 is)The sum of the first n positive integers is \(\displaystyle \frac{n(n+1)}{2}\)

We can rewrite the statement as \(\displaystyle \displaystyle\sum_{i=0}^{n} x^3 =1^3 + 2^3 + ... + n^3 = (1+2+3+...+n)^2=(\frac{n(n+1)}{2})^2\)

Show true for \(\displaystyle n=1\)

\(\displaystyle \displaystyle\sum_{i=0}^{1} x^3 = 0^3 + 1^3 = 1\) and \(\displaystyle 1^2 = 1\)

LHS(Left-hand side) = RHS(Right-hand side), therefore, the statement is true for \(\displaystyle n = 1\)

The induction hypothesis

\(\displaystyle \displaystyle\sum_{i=0}^{k+1} x^3\) \(\displaystyle \stackrel{?}{=}\) is true for some value of \(\displaystyle k≥n\), hence \(\displaystyle k≥1\).(I would write true for n=k)

The induction step

We must show that the statement is true for some value of \(\displaystyle n = k +1\), such that it will also be true for the next value of \(\displaystyle n\).(not very clear)


Assuming that \(\displaystyle \displaystyle\sum_{i=0}^{k} x^3=(\frac{k(k+1)}{2})^2\), we find that

\(\displaystyle \displaystyle\sum_{i=0}^{k+1} x^3= \displaystyle\sum_{i=0}^{k} x^3 + (k+1)^3 = (\frac{k(k+1)}{2})^2 + (k+1)^3= \)

\(\displaystyle \frac{k^2((k+1)^2)}{4} + (k+1)^2 = \frac{k^2(k+1^2) +(4k+4)(k+1)^2}{4} = \frac{(k+2)^2(k+1)^2}{2^2} = (\frac{(k+2)(k+1)}{2})^2 = (\frac{(k+1)((k+1)+1)}{2})^2) \)

The last equation is the same equation as \(\displaystyle (\frac{n(n+1)}{2})^2\) which is based on Theorem1-1, except that n is replaced by (k+1). Hence whenever it is true for all the integers 1,2,...,k, then it is true for the integer k+1.


Very nicely done for a high school student or anyone beginning to do proofs. I did find some errors that are in red above many of which have been already pointed out to you. All in all this is very good.
 
I'd noticed the wrong exponent, too, but forgot to mention it, as it was clearly just a typo.

What I suggest you do is try fixing the errors and improving the wording (perhaps looking at models you were given), and then we can help you finalize it. Some of the wording improvements you need are rather important in the long run, so I do hope we can get to those.
 
I'd noticed the wrong exponent, too, but forgot to mention it, as it was clearly just a typo.

What I suggest you do is try fixing the errors and improving the wording (perhaps looking at models you were given), and then we can help you finalize it. Some of the wording improvements you need are rather important in the long run, so I do hope we can get to those.

Ok, so I've tried to make some changes to the proof. Please let me know if you think that I've made some improvements to the wording. Thanks for the help!
 
Last edited:
Ok, so I've tried to make some changes to the proof. Please let me know if you think that I've made some improvements to the wording. Thanks for the help!

I assume you are planning to post the new proof?
 
No. I just made changes to the proof posted above based on your feedback.

Oh. In my opinion, that is bad form, because it hides the history of the discussion, and people like me don't expect to have to look back there.


Hi. if you find any fault in this proof please let me know since I couldn't find an answer on the back of my math book.

Prove that

\(\displaystyle 1^3 + 2^3 + ... + n^3 = (1+2+3+...+n)^2\)

Statement

Let \(\displaystyle P(n)\) be the statement \(\displaystyle \displaystyle\sum_{i=0}^{n} i^3 =1^3 + 2^3 + ... + n^3 = (1+2+3+...+n)^2\)

The sum of the first \(\displaystyle n\) positive integers is \(\displaystyle \frac{n(n+1)}{2}\) which is a theorem refered to as 1-1.

We can rewrite the statement \(\displaystyle P(n)\) based on theorem 1-1 as \(\displaystyle \displaystyle\sum_{i=0}^{n} i^3 = (\frac{n(n+1)}{2})^2\).

Now we will show that \(\displaystyle P(n)\) is true for all integers \(\displaystyle n \geq 0\)

Base case

\(\displaystyle P(0)\) is the statement \(\displaystyle \displaystyle\sum_{i=0}^{0} i^3 = (\frac{0(0+1)}{2})^2= 0\) which is clearly true.


The induction step

Let \(\displaystyle k \geq 0\) be an integer. Assume (for induction) that \(\displaystyle P(k)\) is true. That means \(\displaystyle \displaystyle\sum_{i=0}^{k} i^3\) \(\displaystyle \stackrel{?}{=}\)\(\displaystyle (\frac{k(k+1)}{2})^2\), is true for \(\displaystyle n=k\).

We will prove that \(\displaystyle P(k+1)\) is true as well. That is, we must prove that \(\displaystyle \displaystyle\sum_{i=0}^{k} i^3 + (k+1)^3= (\frac{(k+1)((k+1)+1)}{2})^2\). To prove this equation, start by adding \(\displaystyle (k+1)^3\)
to both sides of the inductive hypothesis:

\(\displaystyle \displaystyle\sum_{i=0}^{k+1} i^3= (\frac{k(k+1)}{2})^2 + (k+1)^3\). Now, simplifying the right side we get:

\(\displaystyle =\frac{k^2((k+1)^2)}{4} + (k+1)^3 = \frac{k^2(k+1^2) +(4k+4)(k+1)^2}{4} = \frac{(k+2)^2(k+1)^2}{2^2} = (\frac{((k+2)(k+1)=}{2})^2 = (\frac{(k+1)((k+1)+1)}{2})^2 \)

Thus \(\displaystyle P(k+1)\) is true, so by the principle of mathematical induction \(\displaystyle P(n)\) is true for all integers \(\displaystyle n \geq 0\)

Q.E.D.

I'll just make every comment I can, some of which will be utterly trivial!

You didn't correct the statement of the theorem itself to say "for all positive integers n", which is required. Later you allow n to be zero; that is not necessary for the theorem as presumably intended. What was the exact wording of the problem as written?

You still show i starting at 0 rather than 1. This has no effect on the truth of what you say, but is technically a different sum than what was shown in the theorem, and could lead to errors in other problems. But if the theorem was for n >= 0, then all this is correct.

For our purposes it is not necessary to name theorem 1-1 (though as an answer in your class, what you originally wrote was fine). We just need to know it was previously proved.

You don't need the question mark on the equal sign in the induction hypothesis, as that is used to say that something is not known to be true, and here it is being assumed.

I would change the line

That is, we must prove that \(\displaystyle \displaystyle\sum_{i=0}^{k} i^3 + (k+1)^3= (\frac{(k+1)((k+1)+1)}{2})^2\).

to

That is, we must prove that \(\displaystyle \displaystyle\sum_{i=0}^{k+1} i^3 = (\frac{(k+1)((k+1)+1)}{2})^2\).

The left side as you wrote it would be better left to the next line.

I can't find anything more to comment on beyond these quibbles. Excellent work!
 
Hi. if you find any fault in this proof please let me know since I couldn't find an answer on the back of my math book.

Prove that

\(\displaystyle 1^3 + 2^3 + ... + n^3 = (1+2+3+...+n)^2\)

Statement

Let \(\displaystyle P(n)\) be the statement \(\displaystyle \displaystyle\sum_{i=0}^{n} i^3 =1^3 + 2^3 + ... + n^3 = (1+2+3+...+n)^2\)

The sum of the first \(\displaystyle n\) positive integers is \(\displaystyle \frac{n(n+1)}{2}\) which is a theorem refered to as 1-1.

We can rewrite the statement \(\displaystyle P(n)\) based on theorem 1-1 as \(\displaystyle \displaystyle\sum_{i=0}^{n} i^3 = (\frac{n(n+1)}{2})^2\).

Now we will show that \(\displaystyle P(n)\) is true for all integers \(\displaystyle n \geq 0\)

Base case

\(\displaystyle P(0)\) is the statement \(\displaystyle \displaystyle\sum_{i=0}^{0} i^3 = (\frac{0(0+1)}{2})^2= 0\) which is clearly true.


The induction step

Let \(\displaystyle k \geq 0\) be an integer. Assume (for induction) that \(\displaystyle P(k)\) is true. That means \(\displaystyle \displaystyle\sum_{i=0}^{k} i^3\) \(\displaystyle \stackrel{?}{=}\)\(\displaystyle (\frac{k(k+1)}{2})^2\), is true for \(\displaystyle n=k\).

We will prove that \(\displaystyle P(k+1)\) is true as well. That is, we must prove that \(\displaystyle \displaystyle\sum_{i=0}^{k} i^3 + (k+1)^3= (\frac{(k+1)((k+1)+1)}{2})^2\). To prove this equation, start by adding \(\displaystyle (k+1)^3\)
to both sides of the inductive hypothesis:

\(\displaystyle \displaystyle\sum_{i=0}^{k+1} i^3= (\frac{k(k+1)}{2})^2 + (k+1)^3\). Now, simplifying the right side we get:

\(\displaystyle =\frac{k^2((k+1)^2)}{4} + (k+1)^3 = \frac{k^2(k+1^2) +(4k+4)(k+1)^2}{4} = \frac{(k+2)^2(k+1)^2}{2^2} = (\frac{((k+2)(k+1)=}{2})^2 = (\frac{(k+1)((k+1)+1)}{2})^2 \)

Thus \(\displaystyle P(k+1)\) is true, so by the principle of mathematical induction \(\displaystyle P(n)\) is true for all integers \(\displaystyle n \geq 0\)

Q.E.D.



You are getting there. Do not expect your proof to be perfect initially. A few years from now you will look back on your proofs and will see immediate improvements.

if you find any fault in this proof please let me know since I couldn't find an answer on the back of my math book. Since there is no one way to do a proof, the solution in the back of the book will not be unique. At some point you need to develop your own style of writing a proof.
 
Prove that:


\(\displaystyle 1^3 + 2^3 + ... + n^3 = (1+2+3+...+n)^2\). By using the following theorem: The sum of the first \(\displaystyle n\) positive integers is \(\displaystyle \frac{n(n+1)}{2}\) for \(\displaystyle n \geq 1\)


Statement


Let \(\displaystyle P(n)\) be the statement \(\displaystyle \displaystyle\sum_{i=1}^{n} i^3 =1^3 + 2^3 + ... + n^3 = (1+2+3+...+n)^2\)


We can rewrite the statement \(\displaystyle P(n)\) as \(\displaystyle \displaystyle\sum_{i=1}^{n} i^3 = (\frac{n(n+1)}{2})^2\).


I intend to prove that \(\displaystyle P(n)\) is true for all integers \(\displaystyle n \geq 1\)


Base case


\(\displaystyle P(0)\) is the statement \(\displaystyle \displaystyle\sum_{i=1}^{0} i^3 = 0\) which is clearly true.


The induction step


Let \(\displaystyle k \geq 0\) be an integer. Assume (for induction) that \(\displaystyle P(k)\) is true. That means


\(\displaystyle \displaystyle\sum_{i=1}^{k} i^3=(\frac{k(k+1)}{2})^2\), is true for \(\displaystyle n=k\).


Now we will prove that \(\displaystyle P(k+1)\) is true as well.


That is, we must prove that \(\displaystyle \displaystyle\sum_{i=1}^{k+1} i^3 = (\frac{(k+1)((k+1)+1)}{2})^2\).


To prove this equation, start by adding \(\displaystyle (k+1)^3\)to both sides of the inductive hypothesis:


\(\displaystyle \displaystyle\sum_{i=1}^{k} i^3 + (k+1)^3 = (\frac{k(k+1)}{2})^2 + (k+1)^3\) By simplifying the right side we get:


\(\displaystyle \frac{k^2((k+1)^2)}{4} + (k+1)^3 = \frac{k^2(k+1^2) +(4k+4)(k+1)^2}{4} = \frac{(k+2)^2(k+1)^2}{2^2} = (\frac{((k+2)(k+1)}{2})^2 = (\frac{(k+1)((k+1)+1)}{2})^2 \)


Thus \(\displaystyle P(k+1)\) is true, so by the principle of mathematical induction \(\displaystyle P(n)\) is true for all integers \(\displaystyle n \geq 1\)


Q.E.D.


There are fewer errors now entirely thanks to Dr.Peterson and Jomo. Thank's for the help!
 
Last edited:
Prove that:


\(\displaystyle 1^3 + 2^3 + ... + n^3 = (1+2+3+...+n)^2\). By using the following theorem: The sum of the first \(\displaystyle n\) positive integers is \(\displaystyle \frac{n(n+1)}{2}\) for \(\displaystyle n \geq 1\)


Statement


Let \(\displaystyle P(n)\) be the statement \(\displaystyle \displaystyle\sum_{i=1}^{n} i^3 =1^3 + 2^3 + ... + n^3 = (1+2+3+...+n)^2\)


We can rewrite the statement \(\displaystyle P(n)\) as \(\displaystyle \displaystyle\sum_{i=1}^{n} i^3 = (\frac{n(n+1)}{2})^2\).


I intend to prove that \(\displaystyle P(n)\) is true for all integers \(\displaystyle n \geq 1\)


Base case


\(\displaystyle P(0)\) is the statement \(\displaystyle \displaystyle\sum_{i=1}^{0} i^3 = 0\) which is clearly true.


The induction step


Let \(\displaystyle k \geq 0\) be an integer. Assume (for induction) that \(\displaystyle P(k)\) is true. That means


\(\displaystyle \displaystyle\sum_{i=1}^{k} i^3=(\frac{k(k+1)}{2})^2\), is true for \(\displaystyle n=k\).


Now we will prove that \(\displaystyle P(k+1)\) is true as well.


That is, we must prove that \(\displaystyle \displaystyle\sum_{i=1}^{k+1} i^3 = (\frac{(k+1)((k+1)+1)}{2})^2\).


To prove this equation, start by adding \(\displaystyle (k+1)^3\)to both sides of the inductive hypothesis:


\(\displaystyle \displaystyle\sum_{i=1}^{k} i^3 + (k+1)^3 = (\frac{k(k+1)}{2})^2 + (k+1)^3\) By simplifying the right side we get:


\(\displaystyle \frac{k^2((k+1)^2)}{4} + (k+1)^3 = \frac{k^2(k+1^2) +(4k+4)(k+1)^2}{4} = \frac{(k+2)^2(k+1)^2}{2^2} = (\frac{((k+2)(k+1)}{2})^2 = (\frac{(k+1)((k+1)+1)}{2})^2 \)


Thus \(\displaystyle P(k+1)\) is true, so by the principle of mathematical induction \(\displaystyle P(n)\) is true for all integers \(\displaystyle n \geq 1\)


Q.E.D.


There are fewer errors now entirely thanks to Dr.Peterson and Jomo. Thank's for the help!
I intend to prove that P(n) is true for all integers n>=1. I agree with this but you then started showing that P(0) is true. I do not think that you even have that correctly. But more importantly you need to show P(n) is true for n=1.[FONT=MathJax_Main]


[/FONT]By the way at your stage NOTHING is clearly true. You need to prove every little detail. This is how my 1st proof professor treated me and this is how I will treat you. You needed to show that both sides yielded 0 when n=0 (well actually you need to show that both sides are equal for n=1).

This was an excellent proof. If I graded it I would just take off some points for you (not!) showing that P(0) was true instead of P(1). I hate grading proof because they are so hard to grade. I can see myself taking off 20% from your grade since your mistake was a MAJOR part of an induction proof. But then again you are a new student of doing proofs so I can also see myself being lient and just take off 5 points. That is a BIG range I know.
 
I intend to prove that P(n) is true for all integers n>=1. I agree with this but you then started showing that P(0) is true. I do not think that you even have that correctly. But more importantly, you need to show P(n) is true for n=1.[FONT=MathJax_Main]


[/FONT]By the way, at your stage, NOTHING is clearly true. You need to prove every little detail. This is how my 1st proof professor treated me and this is how I will treat you. You needed to show that both sides yielded 0 when n=0 (well actually you need to show that both sides are equal for n=1).

This was an excellent proof. If I graded it I would just take off some points for you (not!) showing that P(0) was true instead of P(1). I hate grading proof because they are so hard to grade. I can see myself taking off 20% of your grade since your mistake was a MAJOR part of an induction proof. But then again you are a new student of doing proofs so I can also see myself being lenient and just take off 5 points. That is a BIG range I know.

Prove that:


\(\displaystyle 1^3 + 2^3 + ... + n^3 = (1+2+3+...+n)^2\). By using the following theorem: The sum of the first \(\displaystyle n\) positive integers is \(\displaystyle \frac{n(n+1)}{2}\) for \(\displaystyle n \geq 1\)


Statement


Let \(\displaystyle P(n)\) be the statement \(\displaystyle \displaystyle\sum_{i=1}^{n} i^3 =1^3 + 2^3 + ... + n^3 = (1+2+3+...+n)^2\)


We can rewrite the statement \(\displaystyle P(n)\) as \(\displaystyle \displaystyle\sum_{i=1}^{n} i^3 = (\frac{n(n+1)}{2})^2\).


I intend to prove that \(\displaystyle P(n)\) is true for all integers \(\displaystyle n \geq 1\)


Base case


\(\displaystyle P(1)\) is the statement \(\displaystyle \displaystyle\sum_{i=1}^{1} i^3 = 1^3= 1\) and \(\displaystyle 1^2 = 1\)

LHS(Left-hand side) = RHS(Right-hand side), therefore, the statement is true for \(\displaystyle n = 1\)


The induction step


Let \(\displaystyle k \geq 1\) be an integer. Assume (for induction) that \(\displaystyle P(k)\) is true. That means


\(\displaystyle \displaystyle\sum_{i=1}^{k} i^3=(\frac{k(k+1)}{2})^2\), is true for \(\displaystyle n=k\).


Now we will prove that \(\displaystyle P(k+1)\) is true as well.


That is, we must prove that \(\displaystyle \displaystyle\sum_{i=1}^{k+1} i^3 = (\frac{(k+1)((k+1)+1)}{2})^2\).


To prove this equation, start by adding \(\displaystyle (k+1)^3\)to both sides of the inductive hypothesis:


\(\displaystyle \displaystyle\sum_{i=1}^{k} i^3 + (k+1)^3 = (\frac{k(k+1)}{2})^2 + (k+1)^3\) By simplifying the right side we get:


\(\displaystyle \frac{k^2((k+1)^2)}{4} + (k+1)^3 = \frac{k^2(k+1^2) +(4k+4)(k+1)^2}{4} = \frac{(k+2)^2(k+1)^2}{2^2} = (\frac{((k+2)(k+1)}{2})^2 = (\frac{(k+1)((k+1)+1)}{2})^2 \)


Thus \(\displaystyle P(k+1)\) is true, so by the principle of mathematical induction \(\displaystyle P(n)\) is true for all integers \(\displaystyle n \geq 1\)\(\displaystyle \quad\Box\)
 
Last edited:
Prove that:


\(\displaystyle 1^3 + 2^3 + ... + n^3 = (1+2+3+...+n)^2\). By using the following theorem: The sum of the first \(\displaystyle n\) positive integers is \(\displaystyle \frac{n(n+1)}{2}\) for \(\displaystyle n \geq 1\)


Statement


Let \(\displaystyle P(n)\) be the statement \(\displaystyle \displaystyle\sum_{i=1}^{n} i^3 =1^3 + 2^3 + ... + n^3 = (1+2+3+...+n)^2\)


We can rewrite the statement \(\displaystyle P(n)\) as \(\displaystyle \displaystyle\sum_{i=1}^{n} i^3 = (\frac{n(n+1)}{2})^2\).


I intend to prove that \(\displaystyle P(n)\) is true for all integers \(\displaystyle n \geq 1\)


Base case


\(\displaystyle P(1)\) is the statement \(\displaystyle \displaystyle\sum_{i=1}^{1} i^3 = 1^3= 1\) and \(\displaystyle 1^2 = 1\)

LHS(Left-hand side) = RHS(Right-hand side), therefore, the statement is true for \(\displaystyle n = 1\)


The induction step


Let \(\displaystyle k \geq 1\) be an integer. Assume (for induction) that \(\displaystyle P(k)\) is true. That means


\(\displaystyle \displaystyle\sum_{i=1}^{k} i^3=(\frac{k(k+1)}{2})^2\), is true for \(\displaystyle n=k\).


Now we will prove that \(\displaystyle P(k+1)\) is true as well.


That is, we must prove that \(\displaystyle \displaystyle\sum_{i=1}^{k+1} i^3 = (\frac{(k+1)((k+1)+1)}{2})^2\).


To prove this equation, start by adding \(\displaystyle (k+1)^3\)to both sides of the inductive hypothesis:


\(\displaystyle \displaystyle\sum_{i=1}^{k} i^3 + (k+1)^3 = (\frac{k(k+1)}{2})^2 + (k+1)^3\) By simplifying the right side we get:


\(\displaystyle \frac{k^2((k+1)^2)}{4} + (k+1)^3 = \frac{k^2(k+1^2) +(4k+4)(k+1)^2}{4} = \frac{(k+2)^2(k+1)^2}{2^2} = (\frac{((k+2)(k+1)}{2})^2 = (\frac{(k+1)((k+1)+1)}{2})^2 \)


Thus \(\displaystyle P(k+1)\) is true, so by the principle of mathematical induction \(\displaystyle P(n)\) is true for all integers \(\displaystyle n \geq 1\)\(\displaystyle \quad\Box\)
Base case
\(\displaystyle P(1)\) is the statement \(\displaystyle \displaystyle\sum_{i=1}^{1} i^3 = 1^3= 1\) and \(\displaystyle 1^2 = 1\) What is this 12 = 1 all about? Show your work! A proof has to be rock solid. Plug in 1 for n in \(\displaystyle (\frac{n(n+1)}{2})^2\). This is quite important as you must prove your base case. \(\displaystyle (\frac{(1)((1)+1)}{2})^2 = (\frac{(1)(2)}{2})^2= 1^2 = 1\)


The induction step
Let \(\displaystyle k \geq 1\) be an integer. Assume (for induction) that \(\displaystyle P(k)\) is true. I would say let n=k. Assume (for induction) that \(\displaystyle P(k)\) is true
 
Last edited:
Base case
\(\displaystyle P(1)\) is the statement \(\displaystyle \displaystyle\sum_{i=1}^{1} i^3 = 1^3= 1\) and \(\displaystyle 1^2 = 1\) What is this 12 = 1 all about? Show your work! A proof has to be rock solid. Plug in 1 for n in \(\displaystyle (\frac{n(n+1)}{2})^2\). This is quite important as you must prove your base case. \(\displaystyle (\frac{(1)((1)+1)}{2})^2 = (\frac{(1)(2)}{2})^2= 1^2 = 1\)
The induction step
Let \(\displaystyle k \geq 1\) be an integer. Assume (for induction) that \(\displaystyle P(k)\) is true. I would say let n=k. Assume (for induction) that \(\displaystyle P(k)\) is true


Yeah. I figured I would leave it out for others imagination... Ok so I've made the final changes now :)

Prove that:


\(\displaystyle 1^3 + 2^3 + ... + n^3 = (1+2+3+...+n)^2\). By using the following theorem: The sum of the first \(\displaystyle n\) positive integers is \(\displaystyle \frac{n(n+1)}{2}\) for \(\displaystyle n \geq 1\)


Statement


Let \(\displaystyle P(n)\) be the statement \(\displaystyle \displaystyle\sum_{i=1}^{n} i^3 =1^3 + 2^3 + ... + n^3 = (1+2+3+...+n)^2\)


We can rewrite the statement \(\displaystyle P(n)\) as \(\displaystyle \displaystyle\sum_{i=1}^{n} i^3 = (\frac{n(n+1)}{2})^2\).


I intend to prove that \(\displaystyle P(n)\) is true for all integers \(\displaystyle n \geq 1\)


Base case


Show the statement is true for \(\displaystyle n=1\)
\(\displaystyle P(1)\) is the statement \(\displaystyle \displaystyle\sum_{i=1}^{1} i^3 = 1^3= 1\) and \(\displaystyle (\frac{(1)(1+1)}{2})^2 = (\frac{(1)(2)}{2})^2= 1^2 = 1\)

LHS(Left-hand side) = RHS(Right-hand side), therefore, the statement is true for \(\displaystyle n = 1\)


The induction step


Let \(\displaystyle k=n\) be an integer. Assume (for induction) that \(\displaystyle P(k)\) is true. That means


\(\displaystyle \displaystyle\sum_{i=1}^{k} i^3=(\frac{k(k+1)}{2})^2\), is true for \(\displaystyle n=k\).


Now we will prove that \(\displaystyle P(k+1)\) is true as well.


That is, we must prove that \(\displaystyle \displaystyle\sum_{i=1}^{k+1} i^3 = (\frac{(k+1)((k+1)+1)}{2})^2\).


To prove this equation, start by adding \(\displaystyle (k+1)^3\)to both sides of the inductive hypothesis:


\(\displaystyle \displaystyle\sum_{i=1}^{k} i^3 + (k+1)^3 = (\frac{k(k+1)}{2})^2 + (k+1)^3\) By simplifying the right side we get:


\(\displaystyle \frac{k^2((k+1)^2)}{4} + (k+1)^3 = \frac{k^2(k+1^2) +(4k+4)(k+1)^2}{4} = \frac{(k+2)^2(k+1)^2}{2^2} = (\frac{((k+2)(k+1)}{2})^2 = (\frac{(k+1)((k+1)+1)}{2})^2 \)


Thus \(\displaystyle P(k+1)\) is true, so by the principle of mathematical induction \(\displaystyle P(n)\) is true for all integers \(\displaystyle n \geq 1\)\(\displaystyle \quad\Box\)
 
Last edited:
The ideas are fine; but you have not written what you meant in some places.

First, you keep writing \(\displaystyle \displaystyle\sum_{i=0}^{n} x^3\). But that means \(\displaystyle \displaystyle\sum_{i=0}^{n} x^3 =x^3 + x^3 + ... + x^3\) ! Presumably you meant to write \(\displaystyle \displaystyle\sum_{i=0}^{n} i^3 = 0^3 + 1^3 + 2^3 + ... + n^3\).

The base case really should be n = 0, not n = 1, the way you've written the sum; you didn't state the conditions on n for the theorem, so if it starts at n = 1, you really should have written \(\displaystyle \displaystyle\sum_{i=1}^{n} i^3 = 1^3 + 2^3 + ... + n^3\).

Where you say you are stating the induction hypothesis, you are not really stating anything (an equal sign with a question mark doesn't assert anything). But the second line under "The Induction Step" is the induction hypothesis.

I could quibble about other parts of the wording, but I don't want to go too far. The important thing is that you got the math right; it's just the symbols and the words that need improvement ...

This was the most onerous situation and i usually skip this one in any conditions :D LOL
 
Top